Reversing Taylor Series to find the original function

Click For Summary
The discussion revolves around finding the original function from a given Taylor series, specifically the series 1/3 + 2/(3^2) + 3/(3^3) + ... + k/(3^k). The user initially calculates the Riemann sum of k/(3^k) and finds it converges to 3/4, but seeks to confirm this by identifying the function. They recognize that the denominator fits the form x/(1-x) when x=1/3, and after some exploration, conclude that the function f(x) = x/((1-x)^2) provides the correct series expansion. The conversation highlights the process of manipulating series to derive the original function effectively.
delta59
Messages
16
Reaction score
0

Homework Statement




I need to find the convergence a unknown function. Now I know the Taylor series of it which is 1/3+2/(3^2)+3/(3^3+4/(4^4+...+k/(3^k). Which mean I can just take the Riemann sum of k/(3^k) from say 0 to 50 and that would give me 3/4.

However this is not enough I need to confirm the answer by finding the unknown function. Normally this isn't too difficult as I would manipulate the series to get it into some sort of form that is familiar like e^x or cos(x), but this one is throwing me for a loop.

The bottom part (3^k) fits x/(1-x) when x=3 but I am not sure about the top

any help or hints would be great!
 
Physics news on Phys.org
I don't know if i am write or wrong , as i have never dealed with them but can't we write them as [summation of (K-1)/3^K]
 
delta59 said:

Homework Statement




I need to find the convergence a unknown function. Now I know the Taylor series of it which is 1/3+2/(3^2)+3/(3^3+4/(4^4+...+k/(3^k). Which mean I can just take the Riemann sum of k/(3^k) from say 0 to 50 and that would give me 3/4.

However this is not enough I need to confirm the answer by finding the unknown function. Normally this isn't too difficult as I would manipulate the series to get it into some sort of form that is familiar like e^x or cos(x), but this one is throwing me for a loop.

The bottom part (3^k) fits x/(1-x) when x=3 but I am not sure about the top

any help or hints would be great!

Welcome to PF, delta59! :smile:

Perhaps you could start with substituting for instance x=1/3?
What would you get?

Afterward, try to think of defining a suitable function f(x) of which you know what the series sum is if you integrate f(x).
(I'll explain later.)
 
x/(1-x) when x=1/3 gives the correct bottom but then I just need to square the bottom function to get the top
so the f(x)=x/((1-x))^2 gives the expansion of 1/3+2/(3^2)+3/(3^3+4/(4^4+...+k/(3^k)

Thanks bouncing ideas I got now.
 
Yep. That works too! :)
 
Question: A clock's minute hand has length 4 and its hour hand has length 3. What is the distance between the tips at the moment when it is increasing most rapidly?(Putnam Exam Question) Answer: Making assumption that both the hands moves at constant angular velocities, the answer is ## \sqrt{7} .## But don't you think this assumption is somewhat doubtful and wrong?

Similar threads

  • · Replies 27 ·
Replies
27
Views
3K
  • · Replies 12 ·
Replies
12
Views
3K
  • · Replies 10 ·
Replies
10
Views
2K
Replies
6
Views
3K
Replies
5
Views
2K
Replies
2
Views
2K
  • · Replies 1 ·
Replies
1
Views
1K
  • · Replies 11 ·
Replies
11
Views
3K
  • · Replies 3 ·
Replies
3
Views
2K
Replies
10
Views
2K